LSAT set3/quest12

This topic has expert replies
User avatar
Master | Next Rank: 500 Posts
Posts: 114
Joined: Fri Oct 08, 2010 10:35 am
Location: TASHKENT
Thanked: 8 times
Followed by:2 members
GMAT Score:640

LSAT set3/quest12

by hoji » Thu Oct 27, 2011 10:20 pm
Photovoltaic power plants produce electricity from sunlight. As a result of astonishing recent technological advances, the cost of producing electric power at photovoltaic power plants, allowing for both construction and operating costs, is one-tenth of what it was 20 years ago, whereas the corresponding cost for traditional plants, which burn fossil fuels, has increased. Thus, photovoltaic power plants offer a less expensive approach to meeting demand for electricity than do traditional power plants.
The conclusion of the argument is properly drawn if which one of the following is assumed?
(A) The cost of producing electric power at traditional plants has increased over the past 20 years.
(B) Twenty years ago, traditional power plants were producing 10 times more electric power than were photovoltaic plants.
(C) None of the recent technological advances in producing electric power at photovoltaic plants can be applied to producing power at traditional plants.
(D) Twenty years ago, the cost of producing electric power at photovoltaic plants was less than 20 times the cost of producing power at traditional plants.
(E) The cost of producing electric power at photovoltaic plants is expected to decrease further, while the cost of producing power at traditional plants is not expected to decrease.

The OA is diffirent that what i chose, C. i thought that C is the right option because the question asks what is assumed, and then clearly assumption question type. C is defender here, ruling out the possibility which weakens the argument. i am stumped. Please help BTGers...?
I know of no more encouraging fact than the unquestionable ability of a man to elevate his life by conscious endeavor.
Henry David Thoreau.

Sleep GMAT, eat GMAT, beat GMAT!
______________________________
Quant ----> 51
Verbal----> 44+
GMAT ----> 750+

Legendary Member
Posts: 544
Joined: Thu Oct 08, 2009 9:14 am
Location: Pune, India
Thanked: 31 times
Followed by:2 members

by adi_800 » Fri Oct 28, 2011 3:00 am
Tough one... Cannot come down to answer...
Thought of A, then D, then E and finally selected E....

User avatar
Senior | Next Rank: 100 Posts
Posts: 34
Joined: Tue Jun 01, 2010 2:17 am
Thanked: 8 times
GMAT Score:730

by Amiable Scholar » Fri Oct 28, 2011 3:26 am
I would go only with D if the ratio is 10 times instead of 20 times.

is 20 times correct figure here
Amiable Scholar

Start Believing in your potential

Junior | Next Rank: 30 Posts
Posts: 13
Joined: Thu Oct 20, 2011 7:06 pm

by vinayaerostar » Fri Oct 28, 2011 8:24 am
IMO D

Senior | Next Rank: 100 Posts
Posts: 85
Joined: Tue Sep 02, 2008 12:13 am
Thanked: 1 times
GMAT Score:650

by vzzai » Fri Nov 04, 2011 4:07 am
IMO A
Thank you,
Vj

User avatar
Legendary Member
Posts: 1101
Joined: Fri Jan 28, 2011 7:26 am
Thanked: 47 times
Followed by:13 members
GMAT Score:640

by HSPA » Fri Nov 04, 2011 5:16 am
I am with E

Conclusion: cost of producing out of photo voltaic cell is less
E on negation: cost of producing power out of photo cell will 'increase'
First take: 640 (50M, 27V) - RC needs 300% improvement
Second take: coming soon..
Regards,
HSPA.